blob: f8df2db5bac7df117f4b8764248e3560e1fc0023 [file] [log] [blame]
avm999637a521d52021-03-20 18:10:54 +01001\documentclass[11pt,a4paper,dvipsnames]{article}
2\usepackage[utf8]{inputenc}
3\usepackage{lmodern}
4\usepackage{amsmath}
5\usepackage{amsfonts}
6\usepackage{amssymb}
7\usepackage{mathtools}
8\usepackage{parskip}
9\usepackage{xcolor}
10\usepackage{tcolorbox}
11\usepackage{hyperref}
12\usepackage{geometry}
13\usepackage{physics}
14\usepackage{systeme,mathtools}
15\usepackage[usestackEOL]{stackengine}
16\usepackage{scalerel}
17\usepackage{graphicx}
18\usepackage{enumerate}
19\usepackage{tikz}
20\usepackage[labelfont=bf]{caption}
21\usepackage{siunitx}
22\usepackage{cancel}
23\usepackage{fbox}
24\usepackage{multicol}
25\usepackage{amsthm}
26\usepackage[shortlabels]{enumitem}
27\usetikzlibrary{positioning}
28\geometry{top=25mm}
29
30% Plantilla per l'interior d'un conjunt
31\newcommand{\interior}[1]{%
32 {\kern0pt#1}^{\mathrm{o}}%
33}
34
35% Plantilles del notate
36\def\myupbracefill#1{\rotatebox{90}{\stretchto{\{}{#1}}}
37\def\rlwd{.5pt}
38\newcommand\notate[4][B]{%
39 \if B#1\else\def\myupbracefill##1{}\fi%
40 \def\useanchorwidth{T}%
41 \setbox0=\hbox{$\displaystyle#2$}%
42 \def\stackalignment{c}\stackunder[-6pt]{%
43 \def\stackalignment{c}\stackunder[-1.5pt]{%
44 \stackunder[2pt]{\strut $\displaystyle#2$}{\myupbracefill{\wd0}}}{%
45 \rule{\rlwd}{#3\baselineskip}}}{%
46 \strut\kern9pt$\rightarrow$\smash{\rlap{$~#4$}}}%
47}
48
49% Plantilles dels boxes
50%%%% START DEFINING LBOXED, RBOXED %%%%
51\newcommand{\lboxed}[1]{\fbox[blt]{\mathsurround=0pt$\displaystyle#1$}}
52\newcommand{\rboxed}[1]{\fbox[brt]{\mathsurround=0pt$\displaystyle#1$}}
53
54% Plantilles pels problemes
55\newcounter{problem}
56\newcounter{solution}
57
58\newcommand{\green}[1]{\textbf{\color{ForestGreen} #1}}
59
60\newcommand{\separator}{\noindent\hfil\rule{0.75\textwidth}{0.4pt}\hfil}
61
62\newenvironment{Problem}{%
63 \stepcounter{problem}%
64 \begin{tcolorbox}[colback=cyan!10!white,parbox=false]
65 \textbf{Problem \theproblem.}~%
66 \setcounter{solution}{0}}{%
67 \end{tcolorbox}
68}
69
70\newenvironment{FreeProblem}{%
71 \begin{tcolorbox}[colback=cyan!10!white,parbox=false]
72 \setcounter{solution}{0}}{%
73 \end{tcolorbox}
74}
75
76\newcommand\Solution{%
77 \textbf{Solution:}\\%
78}
79
80\newcommand\Context{%
81 \textbf{Context:}\\%
82}
83
84\newcommand\Lema{%
85 \textbf{Lema:} %
86}
87
88\newcommand\Proposition{%
89 \textbf{Proposition:} %
90}
91
92\newcommand\Theorem{%
93 \textbf{Theorem:} %
94}
95
96\newcommand\Proof{%
97 \textbf{Proof:}\\%
98}
99
100\newcommand\QED{\square}
101
102\newcommand\ASolution{%
103 \stepcounter{solution}%
104 \textbf{Solution \thesolution:}\\%
105}
106
107
108\newcommand{\asection}[2]{
109\setcounter{section}{#1}
110\addtocounter{section}{-1}
111\section{#2}
112}
113
114% Tensor commands:
115\newcommand*{\TT}[1]{\bar{\bar{#1}}}
116
117\usepackage{biblatex}
118\addbibresource{references.bib}
119
120\title{Homework 1\\Continuum Mechanics}
121\author{Adrià Vilanova Martínez}
122\date{March 20, 2021}
123
124\begin{document}
125
126\maketitle
127
128\begin{Problem}
129 Consider a passive rotation in 2D -a rotation of rectangular axis with common origin.
130
131 \begin{enumerate}[a)]
132 \item Show that the components of vector $A$ transform according to: \[ \begin{pmatrix}
133 a_1 \\
134 a_2
135 \end{pmatrix} = \begin{pmatrix}
136 \cos \theta & - \sin \theta \\
137 \sin \theta & \cos \theta
138 \end{pmatrix} \begin{pmatrix}
139 a_1' \\
140 a_2'
141 \end{pmatrix} = \TT{B} \begin{pmatrix}
142 a_1' \\
143 a_2'
144 \end{pmatrix} \] where $(a_1, a_2)$ are the components of $A$ in orthonormal basis $\{\hat{e}_1, \hat{e}_2\}$ and $(a_1', a_2')$ are the components of $A$ in orthonormal basis $\{\hat{e}_1', \hat{e}_2'\}$, which is rotated an angle $\theta$ relative to the unprimed basis.
145
146 \item Confirm matrix $\TT{B}$ is orthonormal, that is, that its inverse is equal to its transpose. We could have anticipated that since $\TT{B}$ maps an orthonormal basis into an orthonormal basis, which implies the transformation and thus $\TT{B}$ is orthogonal.
147
148 \item Confirm also that the columns and rows of matrix $\TT{B}$ form an orthonormal basis of $\mathbb{R}_2(\mathbb{R})$.
149
150 This is general: a real $n \cross n$ matrix is orthonormal if and only if its rows and columns each form an orthonormal basis of $\mathbb{R}_n(\mathbb{R})$.
151
152 \item Realize that $\det \TT{B} = 1$, as expected for an orthogonal matrix representing a rotation. In fact, any $2 \cross 2$ orthogonal matrix with determinant 1 corresponds to a rotation (the same applies to 3D). Additionally, any $2 \cross 2$ orthogonal matrix with determinant -1 corresponds to a reflection through a line (in 3D, the statement affirms that a reflection is involder -an orthogonal $3 \cross 3$ matrix with determinant -1 thus corresponds to an improper rotation).
153 \end{enumerate}
154\end{Problem}
155
156\textbf{Solution for a):}
157
158Since we know from affine geometry courses that rotations are linear transformations, in order to check that matrix \[ A := \begin{pmatrix}
159 \cos \theta & - \sin \theta \\
160 \sin \theta & \cos \theta
161\end{pmatrix} \] is the transformation matrix which transforms the representation of vectors in orthonormal basis $E' = \{\hat{e}_1', \hat{e}_2'\}$ to their representation in orthonormal basis $E = \{\hat{e}_1, \hat{e}_2\}$, we know from linear algebra courses that it suffices to show that it correctly maps the vectors which form basis $E'$ to their corresponding images.
162
163We know that $\hat{e}_1' = (0, 1)_{E'}$ is the vector $\hat{e}_1 = (0, 1)_E$ rotated by angle $\theta$. Therefore, \[ \hat{e}_1' = (\cos \theta, \sin \theta)_E \] by using the polar coordinates parametrization of $e_1'$.
164
165Also, we know that $\hat{e}_2'$ is the vector $\hat{e}_2$ rotated by angle $\theta$, and also that since $E$ is a direct orthonormal basis of $\mathbb{R}^2$, vector $\hat{e}_2$ is vector $\hat{e}_1$ rotated by $\frac{\pi}{2}$ radians. Therefore, by the same argument used above, \[ \hat{e}_2' = \left(\cos (\theta + \frac{\pi}{2}), \sin (\theta + \frac{\pi}{2})\right)_E = (- \sin \theta, \cos \theta)_E \] where we've used the trigonometric identities $\cos (x + \frac{\pi}{2}) = - \sin x$ and $\sin (x + \frac{\pi}{2}) = \cos x$.
166
167By calculating $A \hat{e}_1'$ and $A \hat{e}_2'$ we can check that the results are the same as what we have found before.
168
169\textbf{Solution for b):}
170
171\[ \TT{B} \text{ orthonormal} \iff \TT{B}^{-1} = \TT{B}^T \iff \TT{B}^T \TT{B} = \TT{I} \]
172
173Let's calculate $\TT{B}^T \TT{B}$:
174
175\[ \TT{B}^T \TT{B} = \begin{pmatrix}
176 \cos \theta & \sin \theta \\
177 - \sin \theta & \cos \theta
178\end{pmatrix} \begin{pmatrix}
179 \cos \theta & - \sin \theta \\
180 \sin \theta & \cos \theta
181\end{pmatrix} = \]
182\[ = \begin{pmatrix}
183 \cos^2 \theta + \sin^2 \theta & - \sin \theta \cos \theta + \sin \theta \cos \theta \\
184 - \sin \theta \cos \theta + \sin \theta \cos \theta & \cos^2 \theta + \sin^2 \theta
185\end{pmatrix} = \begin{pmatrix}
186 1 & 0 \\
187 0 & 1
188\end{pmatrix} = Id \]
189
190Therefore, $\TT{B}$ is orthonormal.
191
192\textbf{Solution for c):}
193
194Let's show that columns $v_1 = (\cos \theta, \sin \theta)$ and $v_2 = (- \sin \theta, \cos \theta)$ (we assume they are expressed in basis $E$) form an orthonormal basis, by definition of an orthonormal basis:
195\[ \begin{cases}
196 (v_1, v_1) = \cos^2 \theta + \sin^2 \theta = 1 \\
197 (v_1, v_2) = - \sin \theta \cos \theta + \sin \theta \cos \theta = 0 \\
198 (v_2, v_2) = \sin^2 \theta + \cos^2 \theta = 1
199\end{cases} \]
200Therefore, they form an orthonormal basis.
201
202Let's check it for the row vectors $\tilde{v}_1 = (\cos \theta, - \sin \theta)$, $\tilde{v}_2 = (\sin \theta, \cos \theta)$ too:
203\[ \begin{cases}
204 (\tilde{v}_1, \tilde{v}_1) = \cos^2 \theta + \sin^2 \theta = 1 \\
205 (\tilde{v}_1, \tilde{v}_2) = \sin \theta \cos \theta - \sin \theta \cos \theta = 0 \\
206 (\tilde{v}_2, \tilde{v}_2) = \sin^2 \theta + \cos^2 \theta = 1
207\end{cases} \]
208Therefore, $\{\tilde{v}_1, \tilde{v}_2\}$ also form an orthonormal basis.
209
210\textbf{Solution for d):}
211
212\[ \det \TT{B} = \cos^2 \theta + \sin^2 \theta = 1 \]
213
214The fact that any $2 \times 2$ orthogonal matrix with determinant 1 represents a rotation and with determinant -1 represents a reflection through a line (and their equivalent statements in 3D) is proved in \cite{geo}.
215
216\begin{Problem}
217 Consider $\TT{B} = \vec{u} \vec{v}$. Show that $\TT{B} \cdot \vec{w} = \vec{u} (\vec{v} \cdot \vec{w})$.
218
219 We often think of the direct product between vectors in terms of its action on a vector. Note how this way of thinking about it directly shows that, in general, the direct product is not commutative: $\vec{u} \vec{v} \neq \vec{v} \vec{u}$.
220\end{Problem}
221
222\Solution
223
224Using Einstein summation convention:
225
226\[ \TT{B}_{ij} = (\vec{u} \vec{v})_{ij} = u_i v_j \implies (\TT{B} \cdot \vec{w})_i = B_{ij} w_k \delta_{jk} = B_{ij} w_j = u_i v_j w_j \]
227\[ (\vec{v} \cdot \vec{w})_j = v_i w_j \delta_{ij} = v_j w_j \implies (\vec{u} (\vec{v} \cdot \vec{w}))_i = u_i v_j w_j \]
228
229Given that we've shown that each component of both vectors in a specific basis are equal to each other, we have proven that in fact both vectors are equal to each other, thus proving the statement.
230
231\begin{Problem}
232 Consider a second rank tensor $\TT{B}$ written in terms of the $\{ \hat{e}_i, \hat{e}_j \}$ basis. Show that $\hat{e}_i \cdot \TT{B} \cdot \hat{e}_j = B_{ij}$ (the $i, j$ component of tensor $\TT{B}$).
233
234 Note this is the tensor expression analogous to the vector expression giving the $i$ component of, say, vector $u$: $u_i = \hat{e}_i \cdot \vec{u}$. In continuum mechanics, we will ofteh consider the $i, j$ component of the stress tensor; it represents the $i$ component of the force (that is, the force along $\hat{e}_i$) per unit area acting on a surface element oriented along direction $j$ (that is, on a surface element with normal along $\hat{e}_j$).
235
236 Note: In quantum mechanics, we often write this tensor expression using bra-kets: $B_{ij} = \langle \hat{e}_i | \TT{B} | \hat{e}_j \rangle$. In this context, we think of $\TT{B}$ as an operator. The quantity $\langle \hat{e}_i | \TT{B} | \hat{e}_j \rangle$ equals the expectation value of the observable represented by operator $\TT{B}$ in quantum state $| \hat{e}_i \rangle$.
237\end{Problem}
238
239\[ (\hat{e}_i \cdot \TT{B})_m = \delta_{ik} B_{lm} \delta_{kl} = B_{im} \implies \]
240\[ \implies \hat{e}_i \cdot \TT{B} \cdot \hat{e}_j = B_{im} \delta_{nj} \delta_{mn} = B_{ij} \]
241
242\begin{Problem}
243 We have stated in class that the symmetry and antisymmetry of second rank tensors are tensor properties, that is, properties that are independent of coordinate system.
244
245 To show this is the case for the symmetry property, first consider how second rank tensors transform. Then assume the tensor is symmetric in the unprimed basis to then show this is also true in the primed basis.
246\end{Problem}
247
248Let's consider a change of basis for a second rank tensor. If $S$ is the change of basis matrix (from basis $B'$ to $B$) and $A$, $A'$ are the matrix representations of a second rank tensor in basis $B$ and $B'$ respectively, we have that:\cite{multi} \[ A' = S^t A S. \]
249
250Therefore: \[ A'^t = (S^t A^t S)^t = S^t A S = A' \] using the property $(AB)^t = B^t A^t$.
251
252\begin{Problem}
253 Use index notation to show that $\TT{B} : \TT{C} = \Tr(\TT{B} \cdot \TT{C})$, where $\TT{B}$ and $\TT{C}$ are second rank tensors.
254\end{Problem}
255
256\[ \TT{B} : \TT{C} = B_{ij} C_{kl} \delta_{jk} \delta_{il} = B_{ij} C_{ji} \]
257\[ (\TT{B} \cdot \TT{C})_{il} = B_{ij} C_{kl} \delta_{jk} = B_{ij} C_{jl} \implies \Tr(\TT{B} \cdot \TT{C}) = \sum_i (\TT{B} \cdot \TT{C})_{ii} = B_{ij} C_{ji} \]
258
259\begin{Problem}
260 Use the quotient rule to show that torque is a second rank tensor.
261\end{Problem}
262
263In class we saw $\tau_{ij} = r_i F_j - r_j F_i$. The quotient rule for second rank vectors states that if we show that for every vector $v_j$ the quantities $u_i := \sum_j T_{ij} v_j$ are the components of a non-zero vector, then $T_{ij}$ are the components of a second rank tensor. It's sufficient to show this for a unit non-zerovector $\hat{e}$. Using Einstein's summation convention: \[ u_i = T_{ij} \hat{e}_j = (r_i F_j - r_j F_i) \hat{e}_j = r_i (\vec{F} \cdot \hat{e}) - F_i (\vec{r} \cdot \hat{e}) \]
264
265The two last terms are component i of 2 vectors, and so is their difference. Therefore, by the quotient rule, this proves that torque is a second rank tensor.
266
267\newpage
268
269\begin{Problem}
270 Prove that $(\vec{u'} \cross \vec{v'})_\alpha = (\det \TT{A}) a_{\alpha i} (\vec{u} \cross \vec{v})_i$, where $\TT{A} = \{ a_{ij} \}$ is an orthogonal transformation from the unprimed to the primed basis. This shows the cross product between two (polar) vectors is a pseudovector.
271
272 It follows that the scalar $\vec{w} \cdot (\vec{u} \cross \vec{v})$, where $u$, $v$ and $w$ are (polar) vectors, is a pseudoscalar. Show this.
273\end{Problem}
274
275As we saw in class: \[ (\vec{a} \cross \vec{b})_i = \varepsilon_{ijk} a_j b_k, \]
276\[ \varepsilon'_{\alpha \beta \gamma} = \det \TT{A} a_{\alpha i} a_{\beta j} a_{\gamma k} \varepsilon_{ijk} \]
277
278Since $\TT{A}$ is the orthogonal transformation from the unprimed to the primed basis, we have that $\TT{A} \cdot \vec{u} = \vec{u'}$ and $\TT{A} \cdot \vec{v} = \vec{v'}$. This means that: \[ (\vec{u'})_i = (\TT{A} \cdot \vec{u})_i = a_{ij} u_k \delta_{jk} = a_{ik} u_k \] and \[ (\vec{v'})_i = a_{ik} v_k. \]
279
280Then:
281\[ \text{LHS} = (\vec{u'} \cross \vec{v'})_\alpha = (\det \TT{A}) a_{\alpha i} a_{\beta j} a_{\gamma k} a_{\beta m} a_{\gamma n} \varepsilon_{ijk} u_m v_n \]
282We have that columns of orthogonal matrix $\TT{A}$ make an orthonormal triad ($a_{ij} a_{ik} = \delta_{ik}$), so:
283\[ \text{LHS} = (\det \TT{A}) a_{\alpha i} \delta_{jm} \delta_{kn} \varepsilon_{ijk} u_m v_n = (\det \TT{A}) a_{\alpha i} \varepsilon_{imn} u_m v_n = (\det \TT{A}) a_{\alpha i} (\vec{u} \cross \vec{v})_i = RHS \]
284
285\printbibliography
286
287\end{document}